<<

MINISTRY OF PUBLIC HEALTH OF UKRAINE

Department of human resources policy, education and science

Testing Board

Student ID Last name

Variant ______

Test items for licensing examination

Krok 2

STOMATOLOGY

General Instruction

Every one of these numbered questions or unfinished statements in

this chapter corresponds to answers or statements endings. Choose the answer (finished statements) that fits best and fill in the circle with the corresponding Latin letter on the answer sheet.

ББК 54.1я73 УДК 61

Authors of items: Aksonova Ye.A., Amosova L.I., Babenko L.M., Babushkina N.S., Bahlyk T.V., Bas O.A., Beliaiev E.V., Bielikov O.B., Bik Ya.H., Bosa L.F., Chaikivsky R.V., Chernov D.V., Chumachenko V.A., Chyrkin V.I., Chyzhevsky I.V., Derkach L.Z., Dmitriyev M.O., Dorubets A.D., Dubrovina O.V., Dvornyk V.M., Dzetsiukh T.I., Eismund A.P., Fedorovych O.A., Flis P.S., Haiduk R.V., Hanchev K.S., Haranina T.S., Hembarovsky M.V., Herasym L.M., Hirchak H.V., Hladka O.M., Hodovanets O.I., Holik V.P., Holovko N.V., Holubieva I.M., Hordiychuk M.O., Hrad I.V., Hrechko N.B., Hrekuliak V.V., Huliuk A.H., Humetsky R.I., Hurzhiy O.V., Hrynkov Ye.I., Idashkina N.H., Ilenko N.M., Ilnytsky Ya.M., Ivchenko N.A., Karasiunok Ye.O., Karelina L.S., Kaskova L.F., Katurova H.F., Khalmatov B.D., Kharchenko O.I., Kharkov L.V., Kirsanova O.V., Klomin V.A., Konovalov M.F., Kopelian N.M., KosarievaL.I., Kotelevska N.V., Koval O.V., Kril A.Y., Kryzhanivska O.O., Kuz H.M., Kuchyrka L.I., Kyrychenko V.M., Larionov I.M., Lavrovska O.M., Levko V.P., Lokes K.P., Lunhu V.I., Lysiuk S.V., Lytovchenko Yu.O., Makarevych A.Yu., Malakhovska A.O., Mikhalova A.O., Moiseitseva L.O., Morozova M.M., Morozova N.P., Nemish T.Yu., Nesin O.F., Odzhubeiska O.D., Oktysiuk Yu.V., Onyshchenko S.I., Orlovsky V.O., Ozhohan Z.R., Parasochkina V.V., Pasechnyk A.M., Pavelko N.M., Petrushanko V.M., Polyshchuk L.F., Potiyko V.I., Prodanchuk A.I., Pryshko Z.R., Raida A.I., Romanenko I.H., Romankov I.O., Romashkina O.A., Rozumenko O.P., Ruzin H.P., Saiapina L.M., Samsonov O.V., Shakhnovsky I.V., Shcherbyna I.M., Shmat S.M., Shubladze H.K., Shuvalov S.M., Semenova O.O., Sidlak O.Ya., Skakun L.M., Skvortsova I.H., Smahliuk L.V., Stasiuk N.O., Svirchkov V.N., Sydorenko A.Yu., Sydorenko I.V., Sydorova A.I., Teslenko O.I., Tsilenko O.L.,Tyuhashkina Ye.H., Ushych A.H., Vasylenko V.M., Voliak M.N., Volkova O.S., Yermakova I.D., Yeroshenko A.V., Yevtushenko L.H., Zinchenko T.P. and Committees of professional expertise.

Item reviewers. Bezvushko E.V., Bulbyk O.I., Chyzhevsky I.V., Dmytriieva A.A., Fastovets O.O., Gerelyuk V.I., Ilenko N.M., Kaskova L.F., Lungu V.I., Muntian L.M., Novikov V.M., Ostapko O.I., Smagliuk L.V., Solovey S.I., Tril S.I., Tsentylo V.G., Volynets V.M., Volyak M.N.

The book includes test items for use at licensing integrated examination “Krok 2. Stomatology” and further use in teaching.

The book has been developed for students of stomatological faculties and academic staff of higher medical educational establishments.

Approved by Ministry of Public Health of Ukraine as examination and teaching publication based on expert conclusions (Orders of MPH of Ukraine of 14.08.1998 №251, of 27.12.1999 №303, of 16.10.2002 №374, of 29.05.2003 №233).

© Copyright Testing Board. Krok 2 Stomatology (англомовний варiант, iноземнi студенти) 2018 рiк 1

1. A removable full denture for the lower A. Deep jaw is being made for a 75-year-old man. B. Front teeth defects that cannot be corrected Objectively the alveolar process is slightly with fillings atrophied. Herbst tests are performed during C. with deformation fitting of an impression tray. When are and discoloration stretched forwards the tray slips off. Where D. should the tray edge be shortened in this E. Devitalized teeth defects that cannot be case? correctedwithdentalinlays A. From canine to canine on the vestibular 5. A 32-year-old woman needs a denture. On side objective examination the decision was made B. From canine to canine on the lingual side in favor of porcelain-fused-to-metal crown. C. From behind the mandibular tuberosity to What material should be used in this case to the mylohyoid line obtain the impression? D. Along the mylohyoid line E. In the premolar area on the lingual side A. Stomaflex B. Repin 2. A 18-year-old boy complains of bleeding C. Stomalgin and pain in her . The disease onset was D. Stens 4 days ago. Objectively the skin is pale, body E. Orthocor temperature is 38.5oC. Her submandibular lymph nodes on the left are enlarged, painful, 6. Parents of a 6-month-old child complain of non-fused with the surrounding tissues. The their child having a large amount of dental gingival papillae and in the deposit in the oral cavity. Objectively the oral area of 33, 34, 35, 36, and 37 are hyperemic, mucosa is hyperemic, covered in white easily ulcerated, and covered in necrotic deposit. removed deposit that resembles curdled mi- Teeth present with soft . Make lk. What is the causative agent of this disease? the diagnosis: A. Candida fungi A. Necrotizing ulcerative B. virus B. Acute catarrhal gingivitis C. Klebs-Loeffler bacillus C. Chronic hypertrophic gingivitis D. Coxsackie virus D. Chronic catarrhal gingivitis E. Epstein-Barr virus E. - 7. A 55-year-old man complains of heartburn 3. A 60-year-old man complains of stabbing and unpleasant sensations in his . The pain near the root of the tongue on the ri- patient has been suffering from gastritis with ght, which develops during eating, especially low acidity for approximately 5 years. What sour food. Objectively: there is a swelling in changes in the tongue are the most likely to the right submandibular area. On palpati- be detected in this patient? on the is dense and A. Atrophied and smoothed out lingual papi- enlarged. Excretory duct orifice of the right llae submandibular gland is dilated and produces B. Hypertrophic mucopurulent secretion. What is the most li- C. Erosions on the lateral surfaces of the kely diagnosis? tongue A. of the submandibular gland D. Coated tongue B. Calculous of the sublingual E. gland 8. A 10-year-old child is referred by the C. Acute suppurative lymphadenitis orthodontist for extraction of tooth 53. D. Acute Objectively the crown of 53 is retained, the E. Adenophlegmon of the right submandi- tooth is immobile. X-ray of tooth 53 shows bular area root resorption by less than 1/3. Choose the 4. A 19-year-old woman, an actress, complai- best instrument for extraction of tooth 53: ns of discoloration of her left maxillary A. Straight crown forceps central incisor. One year ago the of B. Straight elevator this tooth was removed and the tooth was C. Root bayonet forceps filled. Gradually the tooth assumed grayish D. Crown forceps with S-shaped handles . Objectively the 11 is filled, discolored, E. Beak-shaped root forceps firm, painless on percussion. Deep occlusi- on is observed. What part of the clinical 9. A 24-year-old woman has Richmond crown presentation contraindicates installation of being made to restore the crown of the central an all-porcelain crown? maxillar incisor. The cap is completed. What is the next step of prosthesis-making? Krok 2 Stomatology (англомовний варiант, iноземнi студенти) 2018 рiк 2

A. To fit the cap on the tooth stump and place 14. During preventive examination a 40-year- the post in the root canal old man presents with the following changes: B. To solder the post with the cap marginal gingiva is enlarged, torus-shaped, C. To fit the cap and the post to the tooth root cyanotic, slightly bleeding when touched wi- D. Making of combination dental crown th a dental probe; there is no pain. Staining E. Tooth fixation with cement the gums with Lugol’s iodine solution results in light-brown coloring of mucosa. Make the 10. Removable full are being made diagnosis: for a 65-year-old man. Progenic occlusion is determined. What are the specifics of teeth A. Chronic catarrhal gingivitis placement in case of progenic occlusion of B. Acute catarrhal gingivitis edentulous jaws? C. Exacerbation of chronic catarrhal gingivitis D. Chronic hypertrophic gingivitis A. Superior dental arch is shortened by two E. Generalized periodontitis premolars B. Anterior teeth are placed in direct occlusi- 15. A 4-year-old boy has been diagnosed wi- on th acute purulent periostitis of the upper jaw C. Inferior dental arch is shortened by two originating from the 64 tooth. Choose the premolars optimal treatment tactics: D. Anterior teeth are placed in orthognathic occlusion A. The 64 tooth extraction, periosteotomy, E. Short-bite anterior teeth pharmacotherapy B. The 64 tooth extraction, anti-inflammatory 11. A 53-year-old patient complains of pain pharmacotherapy and clicking in the left temporomandibular C. Endodontological treatment of the 64 joint. Objectively: the face is symmetrical, tooth, anti-inflammatory pharmacotherapy palpation of the lateral pterygoid muscles is D. Endodontological treatment of the 64 painful on the left side. Mouth opening is tooth, periosteotomy reduced. Tomography shows smooth E. Periosteotomy, anti-inflammatory outline of joint surfaces. Which disease of pharmacotherapy those listed below corresponds with this clini- cal presentation? 16. An adolescent complains of reduced and painful mouth opening, difficulties when eati- A. dysfunction ng, and swelling in the left mandibular angle B. Rheumatic arthritis that developed after tooth 37 was extracted 3 C. Deforming arthrosis days ago. Objectively the face is asymmetric D. Acute posttraumatic arthritis due to soft tissue swelling in the area of the E. Joint ankylosis left mandibular angle. Mouth opening is pai- nful and reduced to 2.0 cm. Disturbed occlusi- 12. A 47-year-old man complains of partial on is observed. Palpation of the left mandi- loss of his upper teeth. The patient’s medi- bular angle is painful, the tissues are soft, cal history states loss of teeth due to trauma bone crepitus is detected. ”Indirect load to sustained 3 months ago. 11 and 12 are lost. 13, the chin” symptom is positive in the area 21, and 22 are destroyed by 2/3 and restored of the left mandibular angle. The socket of with fillings. Occlusion is orthognathic. What the extracted tooth is packed with iodoform denture construction would be optimal for gauze. What is the most likely diagnosis? this patient, considering his occupation as a lecturer? A. Left mandibular angle fracture B. Mandibular alveolar fracture A. Porcelain-fused-to-metal dental bridge C. Anterior B. Plastic dental bridge D. Mandibular periostitis on the left C. Clasp-retained (bugel) removable partial E. Odontogenic mandibular denture with attachments D. Removable partial laminar denture for the 17. A 32-year-old man has metallic inlay made upper jaw for him. The denture is being made for tooth E. Swaged-soldered metal dental bridge with 36 with Black’s class I carious cavity. What faceted intermediate part surfaces of the inlay should be filed down and polished before fixing the denture? 13. A 7-year-old boy is diagnosed with epi- demic (). Name the most li- A. Occlusal surface kely complication of this disease: B. Lateral surfaces C. All surfaces A. Orchitis D. Inlay bottom B. Colitis E. Lateral surfaces and inlay bottom C. Dermatitis D. Pneumonia 18. A 12-year-old boy complains of pai- E. Cholecystitis nful and bleeding gums on his upper jaw. Objectively the gingival margin in the area Krok 2 Stomatology (англомовний варiант, iноземнi студенти) 2018 рiк 3 of the 13, 12, 11, 21, 22, and 23 teeth is A. of the right swollen, hyperemic, deformed due to gingi- B. Chronic non-epidemic parotitis val overgrowths. Gingival papillae cover the C. Chronic lymphadenitis of the right parotid crowns by 1/3 of their height, bleed on touch. area Upper front teeth are overcrowded. X-ray D. Mixed tumor of the right parotid gland shows no pathological changes of the peri- E. of the right parotid gland odontium. What drugs should be admini- stered for topical treatment in the first place? 22. Parents of a 3-year-old child report that the child suffers from constant pain in the A. Nonsteroidal anti-inflammatory drugs upper front teeth. Objectively: the coronal B. Sclerosants part of the 61 tooth is gray and decayed. C. Steroidal anti-inflammatory drugs Probing of the root canal orifice is painful and D. Keratoplastic agents accompanied by bleeding. The tooth percussi- E. Cytostatic agents on provokes acute pain. Mucosa is hyperemic, edematic and painful. Palpation in the region 19. A 43-year-old woman complains of of the 61 and 62 teeth reveals a fistula. What mobility and displacement of her upper is the provisional diagnosis? front teeth. Objectively: dental formula is 17 16 15 14 13 12 11 21 22 23 24 25 26 27 A. Exacerbation of 47 46 45 44 43 42 41 31 32 33 34 35 36 37 . B. Acute suppurative periodontitis Teeth 12 11 21 22 are slanted towards C. Acute diffuse the vestibular side, diastema and tremata D. Chronic granulating periodontitis are observed, I-II degree teeth mobility is E. Exacerbation of chronic pulpitis detected. Select the orthodontic appliance 23. Puncture sample taken from a 13-year- for correction of teeth misalignment as a part old child contains giant Reed-Sternberg cells. of complex treatment of : What diagnosis can be confirmed by the cell A. Palatal plate with vestibular arch content of this puncture material? B. Bynin appliance A. Lymphogranulomatosis C. Schwartz appliance B. Tuberculous lymphadenitis D. Katz crown C. Lymphocytic leukemia E. Palatal plate with inclined plane D. Lymph node actinomycosis 20. A 45-year-old man complains of E. Infectious mononucleosis toothache and mobility of his upper front 24. A 25-year-old man complains of genelal teeth. Objectively his dental formula is as malaise, high body temperature, acute gi- follows: 17 16 15 14 13 12 11 21 22 23 24 25 26 27 ngival bleeding, and . 47 46 45 44 43 42 41 31 32 33 34 35 36 37 . He has a history of nosebleeds. Objectively the patient presents with systemic lymphoid Dental cervices of 13 12 11 21 22 are hyperplasia, pallor of skin and mucosa, II-III exposed and demonstrate mobility of the degree hyperplasia of the gingival mucosa, III degree. Mobile teeth are to be extracted hemorrhages into the buccal mucosa, and and immediate denture is to be made for the ulcers covered with gray deposit. What patient. How soon after the teeth extraction examination method would be optimal for should such dentures be inserted? diagnosis-making in this case? A. On the day of teeth extraction A. Complete blood test panel B. In 1-2 days B. Yasynsky test C. In 3-4 days C. Bacterioscopy D. In 5-6 days D. Immunoassay E. In 6-7 days E. Blood glucose test 21. A 56-year-old man complains of swelli- 25. A 19-year-old young man complains of ng and pain in his right parotid area. The constant pain in tooth 22, which intensifies on swelling was noticed 5-6 months ago. Objecti- biting with this tooth, sensation of ”protrudi- vely right-sided paresis of the facial muscles ng” tooth, and upper edema. The pati- can be determined. Palpation reveals there ent has history of upper jaw trauma. Objecti- a modrately painful tuberous tumor fused vely tooth 22 is intact. Vertical percussion with surrounding tissues. In the center of is acutely painful. The upper lip is swollen, the tumor there is an area of softening. mucogingival fold in the area of tooth 22 is Submandibular and cervical lymph nodes on red and painful on palpation. What exami- the right are enlarged and dense. The mouth nation method is necessary for diagnosis- can be opened without restriction. There is making in this case? no saliva outflow from the opening of the ri- ght parotid gland. What provisional diagnosis can be made? Krok 2 Stomatology (англомовний варiант, iноземнi студенти) 2018 рiк 4

A. X-ray A. exudativum B. Dental pulp test B. vulgaris C. Rheodentography C. Acute herpetic D. Transillumination D. Nonacantholytic pemphigus E. Thermometry E. Dermatitis herpetiformis (Duhring’s di- sease) 26. A patient complains of carious cavity in tooth 11. The filling was lost one week ago. 30. A 28-year-old woman complains of persi- The tooth crown is dark, there is residual fi- sting pain in tooth 34, which intensifies on lling material at the bottom of the carious biting. Four days ago arsenic paste was left cavity. Vertical percussion is painless. X-ray in the 34. The patient missed her appoi- shows an oval area of bone tissue resorpti- ntment with the dentist. Objective examinati- on with clear margins, 0.4х0.3 cm in size. The on detected occlusive dressing on the distal root canal is filled by 2/3 of its length. What is masticatory surface of tooth 34, percussion is the most likely diagnosis? acutely painful. What treatment tactics would be the most advisable in this case? A. Chronic granulomatous periodontitis B. Chronic fibrous periodontitis A. Arsenic antidote is placed into the root C. Chronic granulating periodontitis canal under occlusive dressing D. Radicular cyst B. dressing is removed, electrophoresis E. Exacerbation of chronic periodontitis with antidote along the mucogingival fold is prescribed 27. A 28-year-old man is referred for oral C. The root canal is lavaged with antidote, the cavity sanation. On examination there is a tooth remains uncovered filling on the masticatory surface of tooth D. The root canal is lavaged with antidote and 17, percussion is painless. Mucosa in the root filled apex projection of 17 is cyanotic, vasoparesis E. - symptom is positive. X-ray shows foci of bone tissue destruction with fuzzy margins in the 31. A 35-year-old man came to the area of root apices, root canals are not filled. prosthodontic clinic with complaints of teeth What is the most likely diagnosis? mobility on his lower jaw. What type of occlusion stabilization is recommended in A. Chronic granulating periodontitis this case? B. Chronic fibrous periodontitis C. Chronic granulomatous periodontitis A. Arch D. Radicular cyst B. Sagittal E. Chronic fibrous pulpitis C. Frontal D. Parasagittal 28. A 30-year-old woman complains of mi- E. Frontosagittal ld burning sensation in her lower lip and its dryness. She peels skin scales off with 32. A 57-year-old patient complains of tooth her teeth. She has been presenting with this mobility and inability to eat. Objectively: the condition for 10 years. On examination the lower 35, 36, 37,38, 44, 45, 46 and 48 teeth are skin scales are gray and located on the lip missing; the 31, 32, 33, 34, 41, 42, 43, 47 teeth from the Klein’s line to the center of the exhibit grade II mobility, their clinical crowns vermillion border from angle to angle of the are low, tooth equator is not pronounced. mouth. The scales are firmly attached in the What is the optimal denture construction in center and are loose on the periphery. Their this case? forcible removal does not result in erosions. What is the most likely diagnosis? A. Removable cast splint B. Removable partial denture A. Exfoliative C. Kurlyandsky splint bar B. D. Bynin removable splint C. Meteorological cheilitis E. Removable splint with vestibulo-oral clasp D. Allergic contact cheilitis E. Eczematous cheilitis 33. A 16-year-old young man complai- ns of temperature increase up to 38.7oC, 29. A 32-year-old patient presents with o pain when eating and swallowing, foul body temperature of 38.9 C, general fatigue, acrid smell from his mouth. Lymph nodes, impaired speech, inability to eat. This condi- especially cervical ones, are enlarged, mobi- tion has been recurring for the last 4 years in le, and painless. Objectively the patient autumn and spring. There are vesicles and presents with generalized hyperemia of the erosions with grayish fibrinous coating on , multiple petechiae, necrotic the hyperemic and swollen labial and buccal spots, and profuse coating of the anteri- mucosa. Nikolsky’s sign is negative. What is or . Blood test: increased ESR, the most likely diagnosis? marked leukocytosis, monocytosis, atypical mononuclear cells, thrombocytopenia. What is the most likely diagnosis? Krok 2 Stomatology (англомовний варiант, iноземнi студенти) 2018 рiк 5

painful along the whole floor of the carious A. Infectious mononucleosis cavity. Electric pulp test - 50 microamperes. B. Vincent stomatitis (acute necrotizing Thermodiagnosis is painful. Make the di- ulcerative gingivitis) agnosis: C. Herpetic angina D. Monoblastic leukemia A. Chronic fibrous pulpitis E. Acute herpetic stomatitis B. Chronic deep caries C. Chronic fibrous periodontitis 34. A 57-year-old retired man complains of D. Acute deep caries attacks of burning pain and rashes on the E. Chronic gangrenous pulpitis skin of his face and oral mucosa on the right. Anamnesis: a course of radiation therapy for 38. A 35-year-old man complains of thickeni- treatment of gastric disease, past case of chi- ng of his maxillary alveolar process. Prelimi- ckenpox. Objectively: along the third branch nary diagnosis of maxillary radicular cyst was of the trigeminal nerve the skin of the face made. What substance will be obtained as the presents with isolated erosions covered in fi- result of the puncture of the alveolar process brinous coating. There are multiple vesicles in the area of buccal thickening? on the hyperemic and swollen oral mucosa. Right-sided lymphadenitis is observed. What A. Yellowish liquid is the most likely diagnosis? B. Blood C. Pus A. Herpes zoster D. Epithelium B. Neuralgia E. Turbid infiltration C. Murrain D. Acute recurrent herpes 39. Six months ago a 40-year-old man had his E. Neuritis tooth 26 extracted; afterwards his oral cavity and maxillary sinus became communicating 35. A 12-year-old girl has complaint of a cari- and the patient developed the first signs of ous cavity in her tooth. Objectively: there is maxillary sinusitis. What surgical procedure Black’s class 1 carious cavity in the 36 tooth; should be performed in this case? it is localized in the parapulpar dentin; the mouth of the cavity is wide. The dentin is A. Maxillary sinusotomy with simultaneous dense and pigmented. It is sensitive to cold plastic surgery for repair of the fistula stimulus, percussion is painless. What is the B. Caldwell-Luc surgery most likely diagnosis? C. Fistula plication D. Fistula packing with Iodoform gauze A. Chronic deep caries E. Osteotomy of the alveolar process B. Chronic median caries C. Acute deep caries 40. A 46-year-old woman complains of D. Acute median caries bleeding gums, suppuration, teeth mobili- E. - ty. She has been presenting with these signs for 10 years. On examination her upper and 36. A 35-year-old woman has complaints of lower gums are hyperemic, swollen, bleed on cosmetic defects of the front upper teeth touch. In the area of 42, 41, 31, and 32 peri- crowns. The defects have been aggravati- odontal pockets are up to 8 mm deep, contain ng for the last 10 years. The patient suffers purulent discharge; these teeth demonstrate from unpleasant sensations when brushi- mobility of the II degree, other teeth present ng her teeth, and when chemical stimuli with mobility of the I degree. In the area of are applied. Objective examination revealed 42, 41, 31, and 32 X-ray shows interalveolar defects localized in the enamel of the front septa resorption by 1/2 of the root length and upper teeth vestibular surface. The defects signs of osteoporosis. What is the most advi- are oval, saucer-shaped, and have clear margi- sable method of surgical treatment in this ns. Response to probing and cold stimuli was case? positive. Make the diagnosis: A. Osteoplasty A. Enamel erosion B. Curettage B. Enamel hypoplasia C. Gingivotomy C. Cuneiform defect D. Gingivectomy D. Chemical necrosis of the tooth E. Flap surgery E. Hyperesthesia of tooth hard tissues 41. A 40-year-old man had his root canal of 37. A patient complains of dull ache in the the34toothfilled due to chronic fibrous peri- 16 tooth, which occurs during eating cold odontitis. Soon the treated place became pai- food. Previously the tooth had been filled nful. On X-ray the root canal of the 34 tooth due to deep caries, the filling was lost 1 year is filled to the root apex. What tactics should ago. Objectively: a deep carious cavity that the dentist choose to manage the pain? does not communicate with the tooth cavity is present; percussion is painless, probing is Krok 2 Stomatology (англомовний варiант, iноземнi студенти) 2018 рiк 6

A. To prescribe physiotherapeutic procedures her. What material would be optimal for this B. To rinse with splint? C. To make insicion along the mucogingival fold A. Cobalt nickel chromium alloy D. To provide conduction B. ”EI-95” alloy E. To provide infiltration anesthesia C. Stainless steel D. ”PD-250” alloy (silver palladium alloy) 42. A 49-year-old man was diagnosed wi- E. Gold alloy of 900 millesimal fineness th recurrence of lower lip cancer two years after he had undergone radiation therapy. 47. A 4-year-old practically healthy child Objectively in the area of his lower right came for oral cavity sanation. Objectively on lip there is a 1x2 cm in size with the masticatory surface of 75 there is a cari- an ulcer in its center. In the right submandi- ous cavity within mantle dentin. The cavity bular area there are 2 round, enlarged, dense, is filled with softened dentin. Dentinoenamel painless lymph nodes. What approach to the junction is painful on probing. What material treatment would be optimal in this case? would be optimal for permanent filling? A. Combined treatment A. Glass ionomer cement B. Wedge resection of the lower lip B. Phosphate cement C. Rectangular resection of the lower lip C. Silicophosphate cement D. Trapezial resection of the lower lip D. Composite material E. Vanakh’s operation E. Silicate cement 43. During regular check-up a 6.5-year-old 48. A 7.5-year-old practically healthy child child presents with carious cavity on the di- complains of crown fracture and pain in the stal proximal surface of 65 within mantle upper right incisor. Objectively 2/3 of crown dentin. Cavity walls and bottom are pi- of 11 is absent, the pulp is exposed and red; gmented, dense, painless on probing; there on probing it is acutely painful and bleedi- is no response to cold stimulus; percussion ng; tooth percussion is painful. The trauma is painless. During tooth preparation there occurred 2 hours ago. What would be the is tenderness at the level of dentinoenamel optimal treatment method in this case? junction. What is the most likely diagnosis? A. Vital amputation A. Chronic median caries B. Devital amputation B. Acute median caries C. Vital extirpation C. Chronic deep caries D. Devital extirpation D. Chronic fibrous pulpitis E. Biological approach E. Chronic granulating periodontitis 49. Parents of a 2.5-year-old child complain of 44. After adenotonsillectomia it is necessary gradual destruction of the upper front teeth to break the habit in a 4- of their child for the last several months. year-old child. The orthodontist recommends Objectively there are carious cavities within application of an oral vestibular shield mantle dentin on the contact and vestibular (Kerbitz’ vestibular plate). Vestibular shield surfaces of 52, 51, 61, and 62. The cavities are facilitates training of the following muscle: filled with softened pigmented dentin that can be easily removed with dental excavator. A. Orbicular muscle Make the provisional diagnosis: B. Temporal muscle C. A. Acute median caries D. B. Chronic deep caries E. Medial pterygoid muscle C. Acute deep caries D. Chronic median caries 45. A 30-year-old man presents with fresh E. Chronic superficial caries median mandibular fracture without visible displacement of the fragments. What will be 50. A 49-year-old man complains of progressi- the function of the dental apparatus to be ng reduction of mouth opening, pain on prescribed in this case? the left when swallowing, severe deteriorati- on of his general well-being, temperature A. Fixation increase up to 39.3oC. Destroyed tooth 38 B. Setting presents with acute pain. Objectively the face C. Directing is symmetrical, the submandibular lymph D. Replacement nodes on the left are enlarged and painful E. Formation on palpation. Palpation under the left mandi- bular angle and in the the left retromandi- 46. A 43-year-old woman complains of her bular area provokes sharp pain. Mouth lower teeth mobility. Objectively the teeth opening and movement of the to mobility is of the I-II degree. It is planned to the left are significantly reduced. The left make a full-cast removable occlusal splint for pterygomandibular fold is hyperemic and Krok 2 Stomatology (англомовний варiант, iноземнi студенти) 2018 рiк 7 infiltrated. What is the most likely diagnosis? transport immobilization of the fracture? A. Phlegmon of the A. Patient’s dentures B. Phlegmon of the B. Weber splint C. Phlegmon of the retromandibular area C. Vankevych splint D. Phlegmon of the submandibular space D. Zbarzh apparatus E. Abscess of the sublingual fossa E. Vasiliev splint 51. A 40-year-old man presents with 55. A 57-year-old woman came to a dentist for pathologic teeth grinding caused by their extraction of the 34 tooth due to exacerbati- functional overload due to the loss of many on of chronic periodontitis. What instrument antagonist teeth. With direct occlusion, verti- would be optimal for tooth extraction in the cal grinding of the front teeth resulted given case? in protrusion of the patient’s lower jaw forwards. Interalveolar space is diminished, A. Beak-shaped non-crushing forceps the lower third of the face is shortened. What B. Beak-shaped crushing forceps would be the most advisable treatment in this C. Beak-shaped curved forceps case? D. Straight elevator E. Curved elevators A. Prosthetics that increase interalveolar height 56. A cast clasp-retained (bugel) removable B. Prothetic treatment partial denture is being made for a 58-year- C. Teeth shortening old patient. Impressions are made, centric jaw D. Prosthetic treatment relation is determined, plaster casts are obtai- E. Instrumental surgical treatment ned. What is the next stage? 52. A 7-year-old child complains of pain and A. Examination of the working model with a swelling in the left submandibular region. parallelometer The swelling in this region developed 2 days B. Transfer of denture frame pattern to the ago. Objectively: the child is in a satisfactory working model condition, body temperature is of 37.3oC. C. Wax modelling of the denture frame Face is asymmetrical due to the soft tissue D. Duplication of the working model swelling in the left submandibular region. E. Marking the border seal Palpation reveals a round formation 2x2 cm in size. The formation is mobile, painful, non- 57. A 7-year-old boy underwent fissure seali- fused with the skin. The 74 tooth is discolored, ng in teeth 36 and 46. Fissure sealing would percussion is painful. What is the provisional be most effective: diagnosis? A. Immediately after eruption of the A. Acute serous odontogenic lymphadenitis permanent tooth of the left submandibular region B. After permanent occlusion is formed B. Acute serous nonodontogenic lymphadeni- C. If permanent teeth are affected with caries tis of the left submandibular region D. In 1-2 years after tooth eruption C. Acute suppurative odontogenic E. In 3-4 years after tooth eruption lymphadenitis of the left submandibular 58. A 6-year-old girl took paracetamol to region treat a case of URTI two days ago, whi- D. Phlegmonous adenitis of the right ch resulted in the development of her submandibular region present condition. The disease onset was E. Lateral cervical cyst acute with temperature increase up to o 53. A 48-year-old patient complains of the 39.8 C. Objectively there are cockade-shaped lower jaw teeth mobility. Van Thiel dental maculopapular rashes on her face. The vermi- splint is to be made for prosthodontic lion border is swollen, hyperemic, covered treatment. What construction elements are in massive brown crusts, and presents with supposed to fixitinplace? bleeding cracks. Conjunctivitis is detected. Swollen and hyperemic oral mucosa presents A. Whole piece proximal grip clasps with numerous erosions covered with fibri- B. Full metal crowns nous incrustations; the erosions are sharply C. Wire clasps painful on palpation. What is the most likely D. Parapulpar posts diagnosis? E. Equator crowns A. Stevens-Johnson syndrome 54. A 55-year-old man suffered a blow to the B. Erythema multiforme exudativum frontal mandibular area. He is diagnosed with C. Acute herpetic stomatitis mandibular fracture. Prior to trauma he was D. Chronic recurrent wearing removable dentures (partial laminar E. Pemphigus denture for the lower law and full denture for the upper jaw). What can be used for 59. How often should the dentures be Krok 2 Stomatology (англомовний варiант, iноземнi студенти) 2018 рiк 8 replaced in children during the period of milk covered in scales of varying size. In the angles occlusion according to Ilyina-Markosian? of the mouth there are fissures covered in white coating, the skin is macerated. What A. Every 6-8 months ointment should be prescribed for topical B. Every 8-10 months treatment in the given case? C. Every 10-12 months D. Every 12-16 months A. Clotrimazol E. Every 16 months B. Interferon C. Prednisolone 60. On objective examination a 59-year-old D. Lanolin man with the edentulous mandible presents E. Erythromycin with bone protrusions and mobile areas of the alveolar crest. To ensure proper fixati- 64. A 35-year-old woman complains of lips on of the denture and even load distribution enlargement. The first incident occurred one the following functional impression should be year ago, when she developed lip edema that made: abated quickly, but the lips remained slightly enlarged. Three days ago after overexposure A. Differentiated to cold her lips enlarged again. Objectively: B. Complete anatomical ptosis, upper and lower lips are markedly C. Compression enlarged, more on the left, soft, elastic, and D. Decompression painless on palpation; no impressions on the E. Combined lip surface are left after pressing it with a fi- nger. The tongue is swollen, with tuberous 61. A 28-year-old man complains of pain in surface and folds on its back. What is the the infraorbital and parotid region on the most likely diagnosis? left. On examination: hemorrhage occurs in the lower eyelid and conjunctiva of the left A. Melkersson-Rosenthal syndrome eye, there are signs of crepitation and step B. Miescher’s granulomatous cheilitis deformity of the eyesocket lower edge. The C. Quincke’s edema mouth opens by 1 cm. Make the diagnosis: D. Achard’s syndrome E. Meige’s trophedema A. Zygomatic B. Malar arch fracture 65. A 45-year-old man complains of dryness C. Left articular process fracture and pain in the lower lip. On examination: D. Traumatic arthritis of the temporo- mandi- the lower lip is swollen, dry, covered in small bular joint scales and fissures. In the Klein area (wet-dry E. Hematoma of the infraorbital region line) there are dilated openings of salivatory 62. A child is 8 years old. There are complai- glands observed as red dots producing clear nts of congested upper incisors. Objectively: substance. The lower lip mucosa is lumpy. the first molars closure is of Angle’s I class, What is the most likely diagnosis? frontal is orthognathic. The 12 and A. Glandular cheilitis 22 teeth erupt palatinally with space defici- B. ency of 2/3 of the tooth crown. The 11 and C. Meteorological cheilitis 21 teeth are 10 mm each in cross-section. The D. Eczematous cheilitis child has inherited father’s facial type with E. Exfoliative cheilitis and of the central incisors. Choose the preventive treatment, 66. A 23-year-old man complains of gum considering this hereditary pathology: bleeding when he brushes his teeth or eats solid food. Objectively: the gums of the front A. Hotz serial extraction to reduce the dental lower jaw are hyperemic, swollen and bleedi- arch ng when palpated. Oral and gingival mucosa B. Jaw expansion to provide the space for the in other areas are not affected. The occlusi- 12 and 21 teeth on is deep. The teeth are firm, except for the C. Massage of the 12 and 21 teeth area to 41 and 31 (degree 1 mobility). X-ray shows stimulate their eruption resorption of the alveolar septum in the area D. Extraction of the 12 and 21 teeth to reduce of the 41, 42, 32, and 31 teeth up to 1/3 of the the dental arch root length. What is the most likely diagnosis? E. Shave off the approximal surfaces of the 11 and 21 to provide the space for the 12 and 22 A. Localized periodontitis teeth B. Generalized periodontitis, initial stage C. Generalized periodontitis, stage I 63. A 38-year-old woman complains of burni- D. Catarrhal gingivitis ng pain in her lips and angles of her mouth, E. Parodontosis, stage I their dryness. Anamnesis states that she has been suffering from diabetes mellitus for 67. A 40-year-old man, a chemical industry the last 8 years. Objectively: the vermilli- worker, notes the sour sensation in his mouth, on border is dry, congestively hyperemic, pain response to thermal and chemical sti- Krok 2 Stomatology (англомовний варiант, iноземнi студенти) 2018 рiк 9 muli. On examination: on the vestibular A. Osteoclastoma surface and cutting edge of the front teeth B. Adamantinoma there are chalky enamel defects with uneven C. Osteoma scalloped margins. Make the diagnosis: D. Follicular cyst E. A. Acidic necrosis of enamel B. Superficial caries 72. A 6.5-year-old child has closed non- C. Enamel hypoplasia (erosive form) pigmented fissures in the first permanent D. Fluorosis (erosive form) molar, which have been revealed duri- E. Median caries ng preventive examination. Enamel transparency is retained, its probing reveals 68. A 42-year-old woman complains of acute no coarseness. Choose the optimal method of lip enlargement, itching, and bursting sensati- treatment in this case: on. She ascribes her condition to introduction of a new lipstick. On examination the lips are A. Non-invasive sealing significantly enlarged and turgid, on palpati- B. Invasive sealing on they are firm, elastic, and painless. Regi- C. Preventive filling onal lymph nodes are without changes. What D. ART technique is the most likely diagnosis? E. Regular medical check-ups A. Allergic contact cheilitis 73. A man complains of gingival pain in B. Meteorological cheilitis his upper left jaw and bleeding that occurs C. Exfoliative cheilitis when he brushes his teeth or eats solid food. D. Glandular cheilitis Objectively on the upper jaw he wears a E. Actinic cheilitis swaged-soldered metal bridge with 14 and 16 as abutment teeth. The crown edge is 69. A 50-year-old patient, an employee of the pushed under the gingival margin by 0.3 print shop, complains of foul smell from his mm. Intermediate part is closely fitted to mouth and excessive salivation. Objectively the gums. The mucosa is hyperemic, swollen, against the background of hyperemic and interdental papillae are smoothed out; slightly swollen gums there is a blue-black touching mucosa with a dental instrument border along the gum margin of the lower provokes bleeding. What medical tactics jaw and upper front teeth. There is a large should the dentist choose in the first place? amount of dental deposit observed. Name the type of stomatitis in this patient: A. Remove the dental bridge B. Refer the patient for cosultation with the A. Lead dental therapist B. Mercury C. Refer the patient for X-ray C. Bismuth D. Refer the patient for clinical blood test D. Catarrhal E. Make metal-fused-to-porcelain dental E. Necrotizing ulcerative bridge 70. A 25-year-old man complains of 74. When checking construction of the incorrectly positioned maxillary left central soldered dental bridge with the 35 and 38 incisor due to trauma sustained 2 months ago. abutment teeth the following was detected: Objectively tooth 21 is rotated around its axis pores in the place where abutment crowns into palatal position. What would be the most and intermediate part are soldered together; advisable treatment method for correction of masticatory cusps are sharply defined; there this defect? is early contact with antagonist teeth; the intermediate part makes tight contact with A. Orthodontic treatment the alveolar process mucosa. How can those B. Surgical treatment flaws be corrected? C. Instrumental surgical treatment D. - A. Dental bridge should be remade E. Splinting followed by prosthetic treatment B. Intermediate part should be corrected C. Masticatory surface should be corrected, 71. A 44-year-old woman complains of the and soldered places - polished face swelling in the right lower jaw area and D. Masticatory surface should be corrected, teeth mobility. Objectively: soft tissues are and the height of the flushing part is to be without changes, the regional lymph nodes increased up to 2 mm cannot be palpated. The alveolar process and E. Tooth-antagonists should be shaved off the body of the lower jaw near the 46, 47, and 48 teeth are thickened, painless when 75. A 50-year-old man was diagnosed with palpated, and lumpy. The teeth in the thi- sialolithiasis with the stone ckened area are mobile. Puncture consists located deep within the salivary gland. of brown fluid without cholesterol crystals. Choose the optimal treatment tactics: What is the provisional diagnosis? Krok 2 Stomatology (англомовний варiант, iноземнi студенти) 2018 рiк 10

A. Submandibular gland excision A. Plaster B. Radiation therapy B. Dentafol C. Sclerotherapy C. Stens D. Removal of the sialolith while retaining the D. Stomaflex gland E. Repin E. Conservative pharmacotherapy 81. A 19-year-old young man complains 76. Due to lack of timely specialized of cosmetic defect of all his teeth, which treatment a 44-year-old man presents wi- developed immediately after the teeth erupti- th incorrectly healed displaced mandibular on. Objectively on the vestibular and masti- fracture. Objectively the lower jaw narrows catory surfaces of all patient’s teeth there sharply, vestibular cusps of the lower teeth are enamel defects, tooth crowns present contact with oral cusps of the upper teeth. with dark brown discoloration. Percussion The patient declined surgical treatment. and probing are painful. In this area fluori- What treatment tactics should be chosen by de levels in water are 2.6 mg/L. Make the the dentist in this case? provisional diagnosis: A. Make a prosthesis with double dentition A. Fluorosis B. Make a non-removable dental bridge with B. Systemic hypoplasia movable joint C. Chronic initial caries C. Correct the by filing down the D. Enamel erosion patient’s teeth E. Chronic superficial caries D. Correct the malocclusion via instrumental surgical method 82. A 14-year-old teenager complains of dry E. Make a dentogingival laminar denture and chapped lips especially in autumn and winter. Objetively the vermillion border is 77. A 57-year-old man presents with habi- dry, infiltrated, and covered in numerous tual mandibular dislocation. To reduce mouth scales. Skin of the lips is dense, pigmented, opening, Yadrova apparatus was made. How with pronounced pattern, peeling, and radial long should the treatment last in this case? cracks. At the same time the skin of the face is dry, lichenified, and excoriated. What is the A. 3 months provisional diagnosis? B. 6 months C. 9 months A. Atopic cheilitis D. 12 months B. Exfoliative cheilitis E. 18 months C. Actinic cheilitis D. Meteorological cheilitis 78. An HIV-infected patients needs a dental E. Allergic contact cheilitis prosthesis. The dentist plans to make dental bridges for this patient. How should the 83. A 56-year-old patient suffering from instruments be processed after the appoi- exacerbation of schizophrenia has been ntment? hospitalized in an oral in-patient department with a diagnosis of the displaced mandibular A. According to the special scheme fracture in the area of teeth 34 and 35. What B. According to the usual scheme method of treatment should be prescribed? C. In a hot air sterilizer D. With lysoformin A. Osteosynthesis E. With 3% chloramine solution B. Onearchsmoothdentalbraces C. Full dental splint 79. A 30-year-old man complains of pain in D. Weber splint his front lower teeth, which he attributes to E. Vankevych splint a trauma to the mental region. Objectively: continuous dentition, orthognathic occlusion. 84. During preventive examination a 5-year- X-ray shows a median mandibular fracture. old child was determined to have insufficient What dental splint would be optimal? physiological of the cusps of the deci- duous canines. What treatment tactics should A. Flat occlusal splint the doctor choose? B. Soldered splint on rings C. Cap splint A. To shave off the retained canine cusps D. Weber’s splint B. Medical examination once a month until E. Plastic mouthguard the incisors are replaced C. Medical examination every 6 months until 80. To make the external prosthesis for a 62- the incisors are replaced year-old man it is necessary to obtain a Hi- D. Medical examination every 6 months until ppocrates facial moulage of this patient. What the canines are replaced impression material should be used? E. No medical intervention is necessary 85. A 60-year-old patient has been undergoi- Krok 2 Stomatology (англомовний варiант, iноземнi студенти) 2018 рiк 11 ng the procedure of checking the complete A. Periosteotomy and pharmacotherapy removable dentures construction and fixi- followed by treatment of the causative tooth ng teeth on wax bases. The following flaws B. - have been detected: fissure between the teeth C. Extraction of the causative tooth, physi- on the frontal area and cusp-to-cusp contact otherapy in the lateral area. What mistake had been D. Extraction of the causative tooth, made? pharmacotherapy E. Extraction of the causative tooth, peri- A. Anterior occlusion had been determined osteotomy instead of central one B. Posterior occlusion had been determined 89. A 30-year-old patient complains of pain instead of central one and swelling in the area of the left parotid C. Lateral occlusion had been determined salivary gland, which occurred 7 days after instead of central one he had undergone abdominal cavity surgery. D. Models had been plastered in a wrong way Objectively: body temperature is 39oC, in an occluder reduced mouth opening, dry mouth; when the E. Swabs had been crushed during determi- gland is massaged, there is purulent exudate nation of central occlusion being secreted from its duct. The patient can be diagnosed with the following disease: 86. A 35-year-old man complains of sour sensation in his mouth and front teeth A. Acute non-epidemic parotitis sensitivity to thermal and mechanical sti- B. Acute epidemic parotitis muli. Objective examination revealed visi- C. Phlegmon of submasseteric space ble changes in the enamel of 13, 12, 11, 21, D. Parenchymatous parotitis 22, and 23. The enamel is dull, rough, mi- E. Phlegmon of parotid-masticatory region ssing on the cutting edge. Probing of the vestibular surface of these teeth is painful, 90. A 45-year-old man presents with facial response to thermal stimuli is positive. The asymmetry due to a dense isolated infiltrati- patient’s medical record states his occupati- on in his right buccal area; the skin over the on in industrial production of inorganic acids. infiltration is cyanotic, thinned out; in the What is the most likely diagnosis? center of the infiltration there is a fistula. In theoralcavitythecrownof46isdestroyed A. Necrosis of dental hard tissues by 2/3, along the mucogingival fold the band B. Fluorosis connecting the tooth with the fistula can be C. Enamel erosion palpated. Make the diagnosis: D. Pathologic teeth grinding E. Enamel hypoplasia A. Migrating facial granuloma B. Chronic mandibular osteomyelitis 87. A 55-year-old man was delivered into the C. furuncle hospital with bilateral mandibular fracture D. Odontogenic lymphadenitis within the dentition. Objectively teeth 34, 35, E. Actinomycosis 36, 45, and 46 are missing. Lower incisors are mobile (I-II degree). Fragment displacement 91. A 53-year-old patient complains of an is insignificant. What splint should be made ulcer on the lateral surface of the tongue. The forthispatient? ulcer appeared 6 months ago as the result of a trauma caused by sharp tip of the 37 tooth A. Weber metal crown. A dentist replaced the crown B. Vankevych with the one of better quality and prescribed C. Tigerstedt keratoplastic drugs. Despite these measures D. Flat occlusal splint the ulcer continues to grow. Lately there has E. Gunning-Port been pain observed during talking, chewi- ng, swallowing; sometimes the pain irradi- 88. A 30-year-old patient is diagnosed with ates to the pharynx. Objectively on the lateral acute suppurative odontogenic periostitis of surface of the tongue there is a painful ulcer the upper left jaw originating from tooth 23. with uneven raised dense margins and lumpy The crown of 23 on the left is destroyed with floor with grayish necrotic coating. What is caries by 1/3. Teeth 22 and 24 are intact. Spot- the most likely diagnosis? film X-ray shows widening of the periodontal fissure of 23. What treatment would be the A. Cancer of the lateral surface of the tongue most advisable in this case? B. Trophic ulcer C. Traumatic ulcer D. Vincent’s necrotizing ulcerative stomatitis E. Tuberculous ulcer 92. A 13-year-old child complains of peri- odical gingival bleeding during teeth brushi- ng, which has been observed for half a year. Objectively the gingival mucosa in the frontal Krok 2 Stomatology (англомовний варiант, iноземнi студенти) 2018 рiк 12 mandibular area presents with congestive A. Calcium hydroxide paste hyperemia and edema. Decay-missing-filled B. Zinc phosphate cement (DMF) index equals 4. Oral cavity hygiene is C. Resorcinol-formalin paste unsatisfactory. In this case it is necessary to D. Glass ionomer cement recommend the patient the toothpastes with: E. - A. Herbal extracts 97. The medical station of a regiment received B. Zinc citrate a patient with signs of bilateral mandibular C. Calcium glycerophosphate fracture. What is the main task of first aid in D. Amine fluorides this case? E. Salt additives A. To control shock, bleeding, and asphyxia 93. A 70-year-old man has edentulous maxi- and to provide transport immobilization lla. Objectively maxillary tuberosity and B. To check and correct previously applied alveolar processes are completely atrophi- bandages ed; palatine vault is flat, its mucosal layer is C. To administer analgesics and cardiac medi- moderately pliant. In this case the patient’s cations atrophic edentulous maxilla can be classified D. To clean oral cavity from blood clots, tooth as: shards, and bone fragments E. To provide symptomatic therapy and care A. Schroeder class III B. Keller class III 98. A 35-year-old man has been hospitali- C. Schroeder class II zed into a dentofacial unit with complaints of D. Keller class II mobility of the 38, 37, and 36 teeth and a fi- E. - stulous tract in the socket of the extracted 35 tooth. The condition has been persisting for 3 94. Teeth 71 and 81 erupted in a 6-year-old months. Insertion of a grooved probe into the child, the lower jaw is retrogenic, the is fistulous tract palpated a bared coarse bone flat with pronounced cross-folds. Determine fragment that easily moved under pressure. the condition of the oral cavity: X-ray of the lower jaw demonstrates a focus of bone tissue destruction, with a spot of A. Physiologic dense bone tissue 0.5х0.3 cm in size. Make B. Pathologic the diagnosis: C. Abnormal D. Subcompensated A. Chronic osteomyelitis E. Decompensated B. Acute osteomyelitis C. Exacerbation of chronic osteomyelitis 95. A 12-year-old girl complains of pain in her D. Chronic periostitis mouth that occurs during eating. Accordi- E. Actinomycosis ng to her medical history these symptoms reemerge once or twice per year. Objectively 99. A 10-year-old child complains of persi- on the mucogingival fold there are 3 aphthae sting throbbing pain in tooth 36, whi- 5-7 mm in size, they have yellowish coati- ch appeared one day ago. Hot stimulus ng and inflamed red border and are acutely aggravates the pain, while cold mitigates it painful on touch. Name the most likely di- slightly. Objectively on the masticatory and agnosis: medial surfaces of tooth 36 there is a deep carious cavity non-communicating with the A. Chronic recurrent aphthous stomatitis dental cavity. Probing of the cavity bottom B. Acute herpetic stomatitis and percussion are painful. X-ray shows no C. Chronic recurrent herpetic stomatitis pathologic changes of the . D. Erythema multiforme exudativum What treatment method shold be chosen in E. Toxic-allergic drug-induced stomatitis this case? 96. An 8-year-old girl complains of tooth A. Vital extirpation 21 discoloration and pain response to hot B. Devital extirpation stimulus. Several months ago tooth 21 was C. Vital amputation treated for acute diffuse pulpitis by means of D. Devital amputation vital amputation. Objectively tooth 21 is fi- E. Conservative treatment lled, percussion is painless. X-ray shows the rooth to be formed by 2/3, cortical plate of 100. A 11-year-old child complains of pain in the tooth socket remains intact in the area of the lower right lateral tooth, which occurs root radix. What material should be used for when eating, especially hot food. On the root canal filling in this case? masticatory surface of the 46 tooth there is a large carious cavity filled with softened light- brown dentin. The cavity is located within parapulpar dentin. In the projection of the medial buccal pulp horn the carious cavity communicates with the pulp chamber. Deep Krok 2 Stomatology (англомовний варiант, iноземнi студенти) 2018 рiк 13 probing is painful. Electric pulp test - 60 mi- A. Make a temporary removable denture croamperes. Make the diagnosis: B. Make a clasp-retained (bugel) removable partial denture A. Chronic gangrenous pulpitis C. Make a dental bridge with 12 and 21 as B. Chronic hypertrophic pulpitis abutment teeth C. Acute diffuse pulpitis D. Temporarily refrain from making a denture D. Chronic fibrous pulpitis E. Perform implantation E. Acute focal pulpitis 105. A 23-year-old man complains of acute 101. A 7-year-old practically healthy child was gingival bleeding and unpleasant smell from undergoing the carious cavity preparation of the mouth that appeared 5 days ago. Objecti- tooth 46 due to acute median caries. During vely gingival papillae and marginal gingi- this procedure the mesio-buccal pulp horn va are friable, bright red, swollen, painful, was accidentally exposed. What treatment and bleed profusely on palpation. Gingival would be optimal in this case? pockets are 3 mm deep. X-ray shows marked osteoporosis of the interalveolar septa, peri- A. Biological approach odontal fissure in the apical areas of the B. Devital amputation interalveolar septa is enlarged. Cortical plate C. Devital extirpation is intact. Make the diagnosis: D. Vital amputation E. Vital extirpation A. Acute catarrhal gingivitis B. Acute leukemia 102. A woman complains of pain in her C. Acute necrotizing ulcerative gingivitis gums, unpleasant smell from her mouth, D. Generalized periodontitis, stage II, difficult eating, general weakness, low- exacerbated development grade fever. Objectively her gums are E. Hypovitaminosis C hyperemic, with areas of ulceration, covered in necrotic deposit. Microscopy revealed 106. A 27-year-old patient has been referred fusospirochetosis. Choose the medication for by a prosthodontist for endodontic treatment etiotropic treatment: of the 45 tooth. Objectively: the 45 tooth crown is destroyed; the lateral surface of A. Metronidazole the tongue and the buccal mucosa have B. Keratoline patches of grayish macerated epithelium sli- C. Galascorbin ghtly protruding above the mucosa surface at D. the points of direct contact with the 45 tooth. E. Chymotrypsin The uvula and palatal bars are stagnant- red in colour; has papulae 103. A man complains of gingival bleeding surrounded with red margin and covered that has been persisting for the last 2 years. in grayish epithelium. The submandibular, Objectively he presents with chronic diffuse cervical, supraclavicular, and subclavicular catarrhal gingivitis, teeth mobility is of the lymph nodes are enlarged and painless. What I degree, periodontal pockets are 2-3 mm is the provisional diagnosis? deep with small amount of serous exudate, occlusion is markedly traumatic. X-ray shows A. Secondary syphilis damaged cortical plate, enlarged periodontal B. Chronic recurrent aphthous stomatitis fissure in the apical areas of the interalveolar C. Lupus erythematosus, patch stage septa, osteoporosis, and interalveolar septa D. Soft (leucoplakia mollis) resorption by 1/3 of their height. Make the E. Lichen ruber planus diagnosis: 107. A 6-year-old boy with congenital heart A. Generalized periodontitis, stage I, chronic disease (pulmonary artery stenosis) presents development with suppurative periostitis of the maxilla. B. Chronic catarrhal gingivitis The child needs surgical treatment. What unit C. Parodontosis, stage I should he be referred to? D. Generalized periodontitis, stage I, exacerbated development A. The pediatric maxillofacial unit E. Generalized periodontitis, early stage, B. The out-patient unit, no precautions are chronic development necessary C. The out-patient unit after preliminary 104. A 55-year-old man came to the cardiological treatment prosthodontic clinic to have a denture made D. The cardiology unit for him. Tooth 11 is missing in the pati- E. The out-patient or in-patient unit at the ent. Two days ago he was released from the in-patient unit after a case of myocardial discretion of the child’s parents infarction. What tactics should the dentist 108. Parents of a 3-year-old child complain of choose? food periodically getting into the child’s nasal cavity during feeding. Objectively there is a fissure in the area of the . Make the Krok 2 Stomatology (англомовний варiант, iноземнi студенти) 2018 рiк 14 diagnosis: A. Temporary molars B. Temporary central incisors A. Isolated partial nonunion of the soft palate C. Temporary lateral incisors B. Congenital hidden cleft palate D. Temporary canines C. Isolated nonunion of the hard and soft E. Front teeth palate D. Cleft hard palate 113. Due to trauma of the area of teeth 44 E. Combined cleft palate and 45, a 12-year-old boy suffers from the pathologic displacement of the mandibular 109. A 9-year-old girl complains of persisting alveolar process and rupture of the alveolar pain in tooth 11 that one month ago sustained process mucosa. What additional examinati- a trauma resulting in broken crown. The tooth on is necessary to specify the diagnosis? received no treatment. Objectively the 1/4 of the tooth crown of 11 is broken off, the remai- A. X-ray of the mandible in frontal and lateral ning crown is grayish, the dental cavity is not projections exposed. Percussion is acutely painful. The B. Skull X-ray in axillary projection mucogingival fold is hyperemic and acutely C. X-ray of the mandible in frontal and Parma painful on palpation. Make the diagnosis: projection D. Computed tomography of the mandible A. Acute traumatic periodontitis E. - B. Acute diffuse suppurative pulpitis 114. A 12-year-old child presents with C. Acute suppurative periodontitis o D. Exacerbation of chronic periodontitis temperature 38 C, chills, nausea, vomiti- E. Chronic granulating periodontitis ng, delirium, and weakness. On the middle third of the face there is hyperemia in a 110. A patient complains of fever up to 38oC, butterfly-shaped pattern. Regional lymph headache, pain in the joints anf muscles, vesi- nodes are enlarged and mildly painful. In cles in the oral cavity, mainly in the frontal blood: leukocytes - 12 · 109/L, lymphocytes - part. Eating is sharply painful. For the last 8.0 · 109/L, ESR- 26 mm/hour. What diagnosis several years the disease has been recurring should be made? during wet and windy weather. The patient often suffers from cases of URTI. Objecti- A. Erysipelatous inflammation vely: on the buccal, lingual, labial mucosa B. Facial vein thrombophlebitis there are confluent erosions against the C. Cutaneous actinomycosis erythematous background, with gray-white D. Streptoderma, submandibular coating. There are bloody scabs on the vermi- lymphadenopathy llion border and in the angles of the mouth. E. Acute non-odontogenic maxillary sinusitis Make the diagnosis: 115. X-ray of the patient shows a focus A. Erythema multiforme exudativum of bone destruction 3x4 cm in size in the B. Syphilis mandibular body. The focus is structured C. Acute aphthous stomatitis as numerous small cavities different in size D. Acute necrotizing ulcerative stomatitis and shape and separated by septa. Tumor E. Chronic recurrent aphthous stomatitis puncture yielded brown liquid. What is the most likely diagnosis? 111. A patient with complaints of toothache in the left upper jaw has made an appoi- A. Giant cell tumor of the mandible ntment with a dental clinic. He was diagnosed B. Radicular cyst of the mandible with chronic periodontitis of the 24 tooth. C. Carcinoma of the mandible What kind of anesthesia is necessary for pai- D. Soft odontoma of the mandible nless extraction of the 24 tooth? E. Mandibular ameloblastoma A. Infraorbital and palatinal anesthesia 116. An orthodontist has been addressed by B. Tuberal and palatinal anesthesia parents of a 5-year-old child. The child has the C. Infraorbital and incisor anesthesia 54 tooth extracted, all the other deciduous D. Tuberal and incisor anesthesia teeth are present. The doctor made a thin- E. Surface and tuberal anesthesia wall crown for the 55 tooth with interdental wedge to the 53 tooth. What is the purpose of 112. A 2.5-year-old child is registered for such treatment? regular check-ups with the orthodontist. The I stage of physiogical occlusion development A. Prevention of dentition malformation corresponds with eruption of the following B. Aesthetic restoration group of temporary teeth: C. Restoration of masticatory efficiency D. Acceleration of permanent tooth eruption E. Deceleration of permanent tooth eruption 117. Removable partial dentures for upper and lower teeth are being made for a 45-year- Krok 2 Stomatology (англомовний варiант, iноземнi студенти) 2018 рiк 15 old man. Complete anatomical impressions should the dental surgeon give to the patient were made using ”Ypeen” alginate material. for the procedure of tooth extraction? What should be used for disinfection of obtai- ned impressions? A. Mandibular and buccal anesthesia B. Intraoral infraorbital nerve block A. 2.5% glutaraldehyde with рH- 7.0- 8.7 C. Tuberal anesthesia B. 0.1% desoxone solution D. Mandibular anesthesia C. - E. Mental nerve block D. Phenol solution in proportion 1:20 E. 6% hydrogen peroxide solution 122. A 34-year-old man complains of soft ti- ssues edema in his lower left jaw and fistulae 118. A 48-year-old man complains of gingival in the submandibular area. Teeth 36 and 37 overgrowth (”gums cover the teeth”). The are destroyed. Alveolar mucosa is swollen patient suffers from epilepsy and takes anti- and hyperemic at the level of 36 and 37.X-ray convulsant agents. Objectively gingival papi- detected sequestra in the mandibular body llae are of normal color, dense, with lumpy on the left. What treatment method should surface; they do not bleed on probing and be chosen in this case? cover the lower front teeth up to their incisal surfaces. What is the most likely diagnosis? A. Extraction of teeth 36 and 37 and mandi- bular sequestrectomy A. Hypertrophic gingivitis, fibrous form, B. Extraction of teeth 36 and 37 degree III C. Mandibular sequestrectomy B. Hypertrophic gingivitis, edematous form, D. Puncture of the inflamed area degree III E. Antibacterial treatment C. Gingival fibromatosis D. Hypertrophic gingivitis, fibrous form, 123. A 42-year-old man was delivered to the hospital in the severe condition: inert, body degree II o E. Hypertrophic gingivitis, edematous form, temperature is 39.1 C, there is acutely painful degree II infiltration of the mouth floor and submandi- bular area on the right. The skin over the 119. A 21-year-old man came to the denti- infiltration is turgid and cyanotic. Palpati- st complaining of general weakness, muscle on detects crepitus under the skin. What di- pain, body temperature up to 38.3oC, indi- agnosis can be made in this case? gestion, excessive salivation, and rashes in the oral and nasal cavities, urethra, on the wi- A. Ludwig’s angina (suppurative-necrotic ngs of the nose, and in the interdigital folds. phlegmon of the mouth floor) These symptoms appeared after ingestion of B. Adenophlegmon of the mouth floor milk during the patient’s stay in the village. C. Malignant tumor of the mouth floor What is the most likely diagnosis? D. Actinomycosis of the mouth floor E. Odontogenic phlegmon of the mouth floor A. Murrain B. Herpetic stomatitis 124. After a blow to the temporomandibular C. Herpes zoster joint the patient developed facial hematoma, D. Behcet’s disease the joint is difficult to move, mandibular E. Infectious mononucleosis mobility is reduced. Attempts to open the mouth wide are painful. What examination 120. A 28-year-old man complains of pai- should be performed to make the diagnosis? nless sore in his mouth that persists despi- te the attempts at self-treatment. Objecti- A. Bilateral X-ray of the temporomandibular vely the regional lymph nodes on the left joint with mouth open and closed are enlarged and painless. Mucosa of the left B. Limit the joint mobility cheek presents with round ulcer, 1 cm in di- C. X-ray and consultation with the neurologist ameter, with raised margins and cartilage-like D. Rheoencephalography and consultation infiltration in its basis. The surface of the ulcer with the neurologist is colored meat red and painless on palpation. E. Panoramic dental X-ray What is the most likely diagnosis? 125. A 38-year-old man after a domestic acci- A. Primary syphilis dent complains of pain and mobility of his B. Cancer upper teeth, problems with eating. Objecti- C. Secondary syphilis vely: soft tissues edema. The 11 and 21 teeth D. Lupus vulgaris are displaced towards the palate, mobile E. Decubitus ulcer (II degree), painful on percussion. Mucosa surrounding the affected teeth is hyperemic 121. A 25-year-old woman made an appoi- and swollen. X-ray demonstrates widened ntment with the dental surgeon for oral cavi- periodontal fissure of the 11 and 21. Choose ty sanation. Objectively the crown of tooth 37 the treatment method: is destroyed by 2/3. Gingival mucosa around tooth 37 is without changes. What anesthesia Krok 2 Stomatology (англомовний варiант, iноземнi студенти) 2018 рiк 16

A. Setting of the teeth and their fixation with is NOT a part of typical procedure of tooth a flat occlusal splint extraction with forceps? B. Extraction of the 11 and 21 teeth C. Reimplantation of the 11 and 21 teeth A. Applying tip of forceps jaw to the edge of D. Immobilization or mouthguard alveolar process E. Removal of tooth pulp in the 11 and 21 B. Applying forceps jaw to the tooth teeth C. Pushing forceps jaw to the cementoenamel junction 126. A 4-year-old child has developed acute D. Closure of forceps handles spontaneous pain in the tooth on the lower E. Tooth dislocation and extraction from the right jaw, which aggravates on biting. Objecti- socket vely: in the 85 tooth there is a deep carious cavity non-communicating with the dental 131. A patient needs the 36 tooth extracted. cavity. Probing is sharply painful at all poi- After administering anesthesia the doctor nts of the cavity floor. Painful reaction to cold started applying the elevator. However, water stimulus and percussion is observed; immediately after that the patient suddenly mucosa surrounding the 85 is hyperemic. paled, complained of dizziness, noise, and Submandibular lymphadenitis is detected. blackout and slid down in the chair. What is Make the provisional diagnosis: the most likely diagnosis? A. Acute pulpitis complicated with peri- A. Unconsciousness odontitis B. Anaphylactic shock B. Acute serous periostitis C. Collapse C. Acute serous periodontitis D. Shock D. Acute suppurative pulpitis E. Hypoglycemic coma E. Exacerbation of chronic periodontitis 132. A child is diagnosed with congeni- 127. During Eschler-Bittner test the profile tal cleft in the soft palate and posteri- of a 12-year-old girl with posterior occlusion or part of the hard palate. What type of has shown some improvement. Specify the anesthesia should be given to the patient for condition that resulted in the development of uranostaphyloplasty? posterior occlusion in this patient: A. Intubation narcosis A. Mandibular underdevelopment B. Intravenous narcosis B. Maxillary overdevelopment C. Anesthesia mask C. Mandibular underdevelopment and maxi- D. Infiltration anesthesia llary overdevelopment E. Conduction anesthesia D. Mandibular overdevelopment E. Maxillary underdevelopment 133. A 6-year-old child complains of pain and edema in the upper right jaw, body 128. Parents of an 8-year-old girl complain of temperature up to 37.9oC, and deterioration their child having an aesthetic defect of her of general well-being. Symptom onset was 3 teeth. Objectively the patient’s lower face is days ago. Objectively the face is asymmetric shortened. Her chin protrudes forwards and due to soft tissue edema of buccal and her upper lip is sunken. During teeth closure infraorbital regions on the right. The crown the deep underbite becomes apparent. of 54 is destroyed by 1/2, percussion is pai- Mesio-occlusion is observed in the lateral nful; the tooth previously had been treated areas. Choose the apparatus optimal for the for complicated caries. On the palatine side treatment: of the affected tooth area there is a painful infiltration with fluctuation in its center; the A. Frankel functional regulator - 3 tissues over the infiltration are hyperemic. B. Frankel functional regulator - 2 Make the provisional diagnosis: C. Osadchy apparatus D. Andresen-Haupl activator A. Acute suppurative periostitis of the maxilla E. Frankel functional regulator - 1 originating from tooth 54 B. Acute serous periostitis of the maxilla 129. During or immediately after an injecti- originating from tooth 54 on, certain local complications can develop. C. Acute odontogenic osteomyelitis of the What is NOT one of those complications? maxilla D. Exacerbation of chronic periodontitis of 54 A. Mucosal necrosis E. Chronic odontogenic osteomyelitis of the B. Dermal ischemia maxilla C. Diplopia D. Functional paralysis or paresis of facial 134. A 27-year-old man presents with missi- muscles ng crown of 11. Objectively teeth 21 and 12 E. Damage to a blood vessel by the needle are intact; intraoral spot film X-ray shows the root of 11 to be filled to the apex, no changes 130. What manipulation of those listed below in the periapical tissues, no pathologic mobi- Krok 2 Stomatology (англомовний варiант, iноземнi студенти) 2018 рiк 17 lity. What construction of the denture should socket and in the area of zygomaticoalveolar be recommended for this patient? crest. What is the most likely diagnosis? A. Metal stump inlay with overlaying A. Displaced fracture of the temporal bone porcelain-fused-to-metal crown B. Le Fort II maxillary fracture B. Stump inlay with overlaying swaged crown C. Nondisplaced fracture of the temporal bone C. Stump inlay with overlaying plastic crown D. Le Fort I maxillary fracture D. Stump inlay with overlaying full cast metal E. fracture crown E. Remove the root of 11 and perform 139. A 14-year-old boy complains of rapid implantation wearing-off of tooth crowns. Objectively: tooth crowns are worn-off by 1/3. Enamel 135. A 54-year-old patient complains of easily chips off and is pale gray in color. Make frequent crunching sound in the right the diagnosis: temporomandibular joint, which developed one month ago. In the morning the crunching A. Stainton-Capdepont syndrome is more frequent and decreases towards the B. Dentinogenesis imperfecta evening. Objectively: the face is symmetrical, C. Fluorosis the skin over the joint is without changes, the D. Systemic hypoplasia mouth opens by 2.9 mm. What is the most E. Focal hypoplasia likely diagnosis in this case? 140. A 63-year-old man complains of pain A. Arthrosis in the area of maxillary mucogingival fold B. Acute arthritis caused by using a removable laminar denture. C. Temporomandibular joint dislocation Objectively: in the area of the mucogingi- D. Chronic arthritis val fold there is a trophic ulcer with swollen E. Temporomandibular joint pain dysfunction margins and hemorrhaging floor. Make the syndrome diagnosis: 136. On examination of a 27-year-old pati- A. Denture-related stomatitis ent the tip of the dental probe caught on B. Toxic chemical stomatitis the fissures of the 36, 37, and 38 teeth. C. Toxic infectious stomatitis Margins of the enamel defect are dark, the D. Allergic surface is coarse. Teeth transillumination wi- E. Greenhouse effect th photopolymer lamp revealed the defect to be limited to the enamel. What is the most 141. Objective examination of a 10-year-old likely diagnosis? child revealed slight hyperemia, infiltration, and dryness of the whole surface of the vermi- A. Chronic superficial caries llion border. Architectonics of the lips is di- B. Chronic median caries sturbed. Dryness and contracted sensation C. Acute superficial caries are observed in the lips, especially during cold D. Chronic initial caries seasons. Make the provisional diagnosis: E. Acute initial caries A. Meteorological cheilitis 137. A 68-year-old patient addressed a surgi- B. Atopic cheilitis cal department of a dental clinic for extracti- C. Allergic contact cheilitis on of the 45 tooth. During procedure the D. Exfoliative cheilitis patient developed burning retrosternal pain E. Cheilitis of microbial origin attack irradiating to the left , scapula, . The skin is pale, BP is 140/100 mm 142. A 24-year-old woman made an appoi- Hg, heart rate is rapid. Skin hyperplasia can ntment with the dental surgeon for extraction be observed in the Zakharin-Head’s zones. of tooth 38. What anesthesia should be gi- What emergency condition did the patient ven to the patient for the procedure of tooth develop? extraction? A. Angina pectoris attack A. Torusal B. Heart failure B. Mandibular C. Bronchial asthma attack C. Tuberal D. Hypertensic crisis D. Infiltration E. - E. Plexus 138. After sustained trauma a man developed 143. During application of tuberal anesthesia nose bleeding, reduced mouth opening, the patient developed rapidly increasing ti- sensation of in the right infraorbi- ssue edema and reduced mouth opening. tal region and lower eyelid. Objectively the What resulted in such a condition? face is asymmetric due to concave right temporal region; step deformity symptom is observed in the middle of the lower right eye Krok 2 Stomatology (англомовний варiант, iноземнi студенти) 2018 рiк 18

A. Vascular trauma B. Muscle trauma during anesthesia applicati- A. Collapse on B. Anaphylactic shock C. Nerve trunk trauma C. Allergic response to the anesthetic D. Intolerance to the anesthetic D. Pain shock E. Anaphylactic shock E. Vert i go 144. A woman came to the dental surgeon wi- 149. An ambulance has delivered an 8- th complaints of teeth mobility. After objecti- year-old child to an admission room. An ve examination and X-ray analysis she was oral surgeon has made the following di- diagnosed with generalized periodontitis of agnosis: odontogenic phlegmon of the right stages I and II. Which teeth of those affected submandibular area. What surgical approach by periodontitis should be extracted? would be advicable for surgical treatment of this phlegmon? A. With degrees 2-3 of tooth mobility B. Intact teeth A. Dissection in the submandibular area, C. Teeth with painful percussion parallel to the mandible D. With degree 1 of tooth mobility B. Dissection parallel to the torus mandi- E. Carious teeth bularis C. Dissection around the mandibular angle 145. After the inflammatory process in the D. Dissection along the lower neck fold parotid area a woman developed frequent E. Dissection in the area of pterygomandi- pain attacks resembling electric current in bular fold her face on the right. The attacks last for 15- 20 minutes. The most likely diagnosis is: 150. A 42-year-old man came to the prosthodontics clinic to have dental A. Trigeminal neuralgia prosthesis made for him. Objectively teeth B. Tympanic plexus neuralgia 34, 35, 36, 43, and 16 are absent in this pati- C. Trigeminal neuritis ent. During examination alginate impressions D. Exacerbation of chronic maxillary sinusitis for diagnostic models were obtained. What E. Exacerbation of chronic osteomyelitis should be used for disinfection of obtained impressions? 146. Mother and her 11-year-old daughter came to the medical station. According to the A. 0.5% sodium hypochlorite solution mother her daughter has fallen when playing B. 3% hydrogen peroxide solution outdoors. The doctor determined the patient C. 10% hydrogen peroxide solution to have an isolated mechanical of soft D. Sterillium tissues in her cheek with damaged skin. Make E. 3% sodium hypochlorite solution the diagnosis: 151. A 36-year-old woman needs a dental A. prosthesis. Objectively there is a carious cavi- B. Contusion ty on the mesial masticatory surface of tooth C. Bruice 46, interdental contact is disturbed. Dental D. Hematoma inlay is to be made for this patient. According E. Wound to Black’s classification of dental caries this cavity is class: 147. The patient with shallow vestibule of mouth and edentulous mandible underwent a A. 2 surgery: a mucoperiosteal flap was relocated B. 3 from the alveolar ridge to the body of the C. 4 mandible and fixed with denture acting as a D. 5 bandage. What surgical procedure was used E. 1 for vestibular deepening? 152. A 45-year-old man complains of pain A. Rumpel and crepitation in the temporomandibular B. Kazanjian joint during the movements of the lower C. Thiersch jaw. Objectively: the face is symmetrical, D. Trauner the mouth opens with slight displacement E. Rhermann to the left. Dentition is intact. On occlusi- ography there were detected centric and 148. A 7-year-old girl received conduction eccentric supracontacts. What treatment anesthesia with 2% articaine solution for methods should be applied in the first place? extraction of tooth 16. She has no history of allergies. After receiving anesthesia the pati- ent complained of weakness, she developed skin pallor, cyanosis, and nausea. Her blood pressure dropped significantly. The patient is conscious. What is the provisional diagnosis? Krok 2 Stomatology (англомовний варiант, iноземнi студенти) 2018 рiк 19

A. Selective teeth shaving A. Fixation of the lower jaw and tongue with B. Mouthguard for muscle relaxation standard Entin’s head-chin strap C. Appliances that limit mouth opening B. Fixation of the tongue to the patient’s D. Mouthguards that increase the height of collar central occlusion C. Tracheostomy and artificial pulmonary E. Lower jaw immobilization ventilation D. Removal of foreign bodies from the oral 153. A 45-year-old patient after administrati- cavity on of local anesthesia in preparation for oral E. Excision of injured mucosal flaps surgery has suddenly felt unwell, developed increasing edema of laryngeal mucosa and 157. The hospital received a 19-year-old pati- respiration disorder. The dentist stopped the ent injured in a landmine explosion wi- manipulations in the oral cavity. What type of th an open displaced mandibular fracture asphyxia developed in the patient? in the area of the left mandibular angle and a contused lacerated wound of the A. Stenotic left submandibular area. The patient is in B. Dislocational moderately severe condition, pulse is 80/min., C. Val vul ar blood pressure is 110/80 mm Hg. What D. Obturative anesthesia should be given to the patient for E. Aspiration initial surgical d-bridement? 154. A 19-year-old young man, who was wai- A. Endotracheal anesthesia ting for the appointed time at the denti- B. Intravenous anesthesia st’s, suddenly developed an attack: his face C. Neuroleptanalgesia became purple, bloody foam flowed from D. Conduction anesthesia with premedication his mouth (bitten tongue), pupils were di- E. Conduction anesthesia lated and unresponsive to light, the patient developed first tonic then clonic convulsi- 158. A 5-year-old child developed a ons that stopped spontaneously, after that he hemorrhage after pulp extirpation of 74 due calmed down and fell asleep quickly. What to exacerbated chronic pulpitis. The child happened with the patient? suffers from Von Willebrand disease. What actions should be taken by the dental surgeon A. Epileptic seizure to stop bleeding? B. Sympathoadrenal crisis C. Spasmophilia A. To hospitalize the child to the hematologi- D. Morgagni-Adams-Stokes syndrome cal unit E. Pulmonary embolism B. To hospitalize the child to the maxillofacial unit 155. A victim of a traffic accident was deli- C. To place sutures on the mucosa vered into the admission room. The patient is D. To plug the cavity with epsilon- supine and unconscious. His skin is cyanotic, aminocaproic acid dressing respiration is extremely labored, mucosa is E. To plug the cavity with hemostatic sponge pale, blood clots are accumulated in the oral cavity.The patient is diagnosed with displaced 159. Parents of a 9-year-old child came to the bilateral mandibular fracture. How should dentist complaining that their child presents this patient be transported? with enlarged cervical lymph nodes on the ri- ght. During examination palpation revealed A. In the prone position on a soft stretcher the lymph nodes in the right submandibular, B. Positioned on the side on a rigid stretcher cervical, supraclavicular, and infraclavicular C. In the sitting position with the head thrown areas to be enlarged up to 2-2.5 cm in di- back ameter, painless, non-matted together, and D. In the sitting position with air tube inserted non-fused to the skin (resemble ”potatoes into the upper airways in a sack”). The parents note rapid fatigabi- E. Positioned on the side on a soft stretcher lity and night sweats in their child. What additional examinations should the child be 156. A victim of a traffic accident was deli- referred for? vered into the admission room. The patient is supine and unconscious. His skin is cyanotic, A. Puncture biopsy of the lymph nodes respiration is extremely labored, mucosa is B. Pirquet and Mantoux tests pale, blood clots are accumulated in the oral C. Wassermann test cavity.The patient is diagnosed with displaced D. CT of the cervical spine bilateral mandibular fracture. What measures E. Clinical blood and urine tests should be taken to prevent complications in this case? 160. A 45-year-old patient complains of inabi- lity to properly masticate due to the loss of lateral teeth. The 17, 16, 15, 25, 26, 27, 37, 36, 35, 44, 45, and 46 teeth are missing. The retai- ned teeth exhibit the I-II degree of mobi- Krok 2 Stomatology (англомовний варiант, iноземнi студенти) 2018 рiк 20 lity. The patient is diagnosed with generali- probing causes severe pain, percussion of the zed periodontitis. Kennedy class I dentiti- 17 tooth is painful. X-ray: there is slight wi- on defects are observed. What construction dening of the periodontal fissure near the root would be optimal in the given case? apex. Electric pulp test - 70 microamperes. What final diagnosis can be made? A. Clasp-retained (bugel) removable partial denture with splinting elements A. Chronic gangrenous pulpitis B. Partial laminar denture B. Chronic fibrous pulpitis C. Elbrecht’s dental splint C. Acute purulent pulpitis D. Mamlok’s dental splint D. Chronic fibrous periodontitis E. Cantilever dental bridges E. Exacerbation of chronic fibrous periodonti- tis 161. A 70-year-old patient addressed a hospi- tal with complaints of poorly stabilized 165. Medical committee registers the patients, complete removable dentures of the upper who for a long time lived in an area polluted and lower jaws. What method of artificial with radiation. The patients are advised on teeth arrangement is preferable in making the diet that will quickly purge the body from of a new complete removable denture? radionuclides. The portion of products rich in pectine should be increased in their diet. A. According to individual occlusal curves Name these products: B. According to disocclusal planes C. According to standard occlusal curves A. Fruits and vegetables D. According to spherical occlusal curves B. Meat products E. According to prothetic occlusal planes C. Pasta D. Dairy products 162. A 2.5-year-old child has fever up to E. Baked goods 38.5oC, low appetite, rashes in the oral cavity. The disease onset was 3 days ago. Objecti- 166. A 44-year-old man came to extract vely: the skin of the perioral area is covered in destroyed tooth 24. Objectively his face is scarce vesicles with clear content. Within the symmetrical, the crown of 24 is destroyed by oral cavity on the buccal and lingual mucosa 2/3, percussion is painless. Gingival mucosa there are sharply painful erosions, 2-3 mm surrounding the tooth is unchanged. X-ray in size, with white coating and hyperemic shows enlarged periodontal fissure in the area crown. The gums are swollen, hyperemic. The of the root apex. What is the most likely di- submandibular lymph nodes are enlarged, agnosis? painful on palpation. Make the diagnosis: A. Chronic fibrous periodontitis of 24 A. Acute herpetic stomatitis B. Chronic granulating periodontitis of 24 B. Stevens-Johnson syndrome C. Chronic granulomatous periodontitis of 24 C. Erythema multiforme exudativum D. Exacerbation of chronic periodontitis of 24 D. Stomatitis with the background of infecti- E. Chronic gangrenous periodontitis of 24 ous mononucleosis E. Stomatitis with the background of chi- 167. A 46-year-old man complains of constant ckenpox losing of a filling in his lower right tooth. Objectively: in the 16 tooth on the approxi- 163. A 25-year-old man complains of short- mal masticatory surface there is a defect of term pain in the tooth on the lower ri- crown hard tissues at 1/3. The tooth has no ght jaw during eating sweet, hot, and cold discoloration, percussion is painless. What food. Objectively: in the 36 tooth on the construction should be prescribed? distal surface there is a carious cavity non- communicating with the dental cavity, dentin A. Inlay is softened. Probing of the cavity floor is pai- B. Porcelain-fused-to-metal crown nful, percussion is painless. Electric pulp test C. Semi-crown is 16 microamperes. Make the final diagnosis: D. 3/4 crown E. Plastic crown A. Acute deep caries B. Acute median caries 168. A 10-year-old boy complains of acute C. Pulpal hyperemia pain attacks in the area of his upper left teeth. D. Chronic gangrenous pulpitis The toothache persisted for a night. Objecti- E. Chronic fibrous periodontitis ve examination revealed a carious cavity on the masticatory surface of the 26 tooth within 164. A 30-year-old woman came to the dentist parapulpar dentin. Probing is sharply painful with complaints of uncomfortable sensation at all points of the cavity floor. Markedly of pressure in her upper right tooth, which positive reaction to cold water stimulus is aggravates in response to hot stimulus, and observed. Select the most likely diagnosis: foul smell from the mouth. Objectively: there is a deep carious cavity in the 17 tooth, whi- ch communicates with the tooth cavity. Deep Krok 2 Stomatology (англомовний варiант, iноземнi студенти) 2018 рiк 21

A. Acute diffuse pulpitis patient? B. Acute serous periodontitis C. Acute suppurative pulpitis A. Biological approach D. Acute suppurative periodontitis B. Vital amputation E. Acute local pulpitis C. Vital extirpation D. Devital amputation 169. In a state of inebriation a 36-year-old E. Devital extirpation woman suffered a domestic accident 4 hours ago. She was delivered to the maxillofacial 173. A 37-year-old woman came to the dentist surgery unit. On examination she was provisi- with complaints of brief attacks of toothache onally diagnosed with a deep perforating inci- caused by eating sweets. Objectively there is sed wound of the lateral surface of her face a shallow carious cavity within enamel. On with injured perioral area and parotid gland. probing cavity walls and bottom are coarse; The woung edges are gaping. This gaping is there is no response to thermal stimuli. Make caused by: the diagnosis: A. Location of facial muscles A. Acute superficial caries B. Wound communication with the oral cavity B. Endemic fluorosis C. Specifics of facial innervation C. Enamel hypoplasia D. Location of masticatory muscles D. Acute median caries E. Depth of the wound E. Chronic median caries 170. The maxillofacial surgery unit recei- 174. A 24-year-old patient came to the dentist ved a patient with complaints of inability complaining of chalky lesions on the front to close his mouth. This condition occurred teeth. Objectively teeth 13, 12, 11, 21, 22, when the patient was biting an apple. Objecti- and 23 present with chalky lesions separated vely there is a frightened expression on the by areas of healthy unchanged enamel. Lesi- patient’s face, the mouth is open wide, the on surface is coarse; there is no response to chin is displaced to the left, salivation is thermal stimuli. Childhood years of the pati- observed. Palpation through the external ent were spent in the area with fluorine level acoustic meatus detected no movements of of 1.8 mg/L in drinking water. Make the di- the right articular head. What is the most li- agnosis: kely diagnosis? A. Endemic fluorosis A. Right temporomandibular joint dislocation B. Enamel hypoplasia B. Fracture of the mandibular process C. Enamel hyperplasia C. Acute temporomandibular arthritis D. Acute superficial caries D. Temporomandibular joint pain dysfunction E. Chronic superficial caries syndrome E. Bilateral temporomandibular joint di- 175. A 48-year-old patient has addressed a slocation hospital with complaints of defects in the paragingival area and slight sensitivity to 171. A 19-year-old girl addressed an oncologi- thermal stimuli. Objectively there are hard st with complaints of slowly growing tumor- tissue defects that resemble a wedge with like mass on the tip of her tongue. The mass smooth polished walls on the precervical was first noticed 5-6 years ago. The pati- vestibular surface of the 23 and 24 teeth. ent requested no medical help. Objectively: Thermal test is slightly positive. What is the there is a pale pink round growth with wi- most likely diagnosis? de pedicle on the tongue apex; the growth is painless, elastic; there are no changes of A. Cuneiform defect mucosa surrounding the pedicle. Submandi- B. Enamel necrosis bular lymph nodes cannot be palpated. What C. Acute deep caries kind of tumor is it? D. Enamel erosion E. Endemic fluorosis A. Papilloma B. Atheroma 176. Parents of a 7-year-old child addressed a C. hospital with complaints of their child havi- D. Lipoma ng no permanent teeth in the front area of E. Keratoma the mandible. Anamnesis states that the fi- rst erupted at the age of 172. During carious cavity preparation in a 11 months. Objective clinical examination 20-year-old man, the pulp-chamber floor was revealed the following: appearance is without accidentally perforated and horn of the pulp changes; milk occlusion; there are physiologi- was exposed. On the carious cavity floor there cal diastema and tremata; edge-to-edge inci- is a point-like puncture surrounded with whi- sor contact. What preliminary diagnosis can te predentin. Pink pulp can be seen through be made according to Kalvelis classification? the perforation, pulp probing is acutely pai- nful. What treatment should be given to the Krok 2 Stomatology (англомовний варiант, iноземнi студенти) 2018 рiк 22

A. Retarded eruption B. Supernumerary tooth A. Mamlok’s dental splint C. Adentia B. Removable segmented splint for the front D. Dystopia teeth E. Hypoplasia C. Cap splint D. Splint with embrasure clasps 177. A 5.5-year-old child is undergoi- E. Semicrown splint ng preventive examination. There are no complaints from the patient. Objectively: in 182. A woman complains of spontaneous the precervical area of buccal surface of the attacks of acute pain, with practically no 75 tooth there is an enamel patch with loss intermissions and irradiation into the temple; of natural glossiness. The enamel surface is cold water slightly mitigates the pain. In tooth coarse, painless on probing. Tooth percussi- 26 examiantion revealed deep carious cavity on is painless. Thermodiagnosis is negative. non-communicating with the dental cavity. Damaged enamel stains with 2% water soluti- Probing of the cavity bottom is acutely pai- on of methylene blue. Make the diagnosis: nful, the tooth is tender on vertical percussi- on. Make the provisional diagnosis regarding A. Acute initial caries tooth 26: B. Enamel hypoplasia C. Fluorosis A. Acute suppurative pulpitis D. Acute superficial caries B. Acute diffuse pulpitis E. Chronic initial caries C. Pulpal hyperemia D. Chronic fibrous pulpitis 178. A 68-year-old man has removable parti- E. Chronic hypertrophic pulpitis al dentures made for him. At the stage of checking the denture construction, occlusal 183. A 78-year-old man complains of a painful contact in the lateral areas is cusp-to-cusp sore in his mouth that has been persisting for and frontal area has sagittal fissure. What mi- 2 months already. The patient is a smoker. stake was made by the doctor? Objectively on the buccal mucosa on the ri- ght there is a shallow ulcer up to 1.5 cm in A. Anterior occlusion is fixed size with lumpy floor and uneven margins. B. Lateral occlusion is fixed There are yellowish granules on its periphery. C. Prothetic plane is designed incorrectly Palpation is painful, the lesion is soft. Regi- D. Height of centric occlusion is understated onal lymph nodes are enlarged, painful, and E. Height of centric occlusion is overstated matted together. What is the provisional di- agnosis? 179. An 80-year-old man needs a removable partial denture for the lower jaw. How should A. Tuberculous ulcer the denture boder be located relative to the B. Cancerous ulcer natural teeth? C. Decubitus ulcer D. Trophic ulcer A. Covers the crowns by 2/3 of their height E. Hard chancre B. Covers the crowns by 1/3 of their height C. Fully covers the crowns 184. A 37-year-old patient has symmetrical D. Reaches dental cervices face; the mucosa in the area of the 12 tooth E. Located below dental cervices root apex projection is pale pink; palpation is painless; the tooth crown is destroyed by 1/3; 180. A 52-year-old woman needs a dental percussion is painless. X-ray: the root canal of prosthesis. Upon objective examination a the12toothisfilled to the apex; granuloma 4 removable partial laminar denture is determi- mm in diameter is located near the root apex. ned to be optimal denture construction in her Choose the method of surgical treatment: case. What material should be applied to the impression tray edges? A. Granuloma removal with root apex resecti- on A. Orthocor B. Root hemisection B. Plaster C. Coronary radicular tooth separation C. Wax D. Root amputation D. Silast E. Tooth extraction E. Stens 185. A 48-year-old man presents with verruci- 181. A 46-year-old man, a teacher, complai- form, dense, gray-white growths on the buccal ns of lower teeth mobility that impedes the mucosa. The growths protrude above the process of biting. Objectively the dentition is neighbouring tissues and are surrounded by uninterrupted, front teeth demonstrate the II keratinized gray-white spots that cannot be degree of mobility. X-ray shows straight and scraped off. Make the provisional diagnosis: filled root canals of 32, 31, 41, and 42. What appliance will stabilize front teeth while retai- ning their aesthetic appearance? Krok 2 Stomatology (англомовний варiант, iноземнi студенти) 2018 рiк 23

A. Verrucous leukoplakia response to thermal stimuli. Percussion of B. Erosive leukoplakia tooth 24 is acutely painful. X-ray shows no C. Bowen’s disease pathologic changes of periapical tissues in D. Erythroplasia of Queyrat the area of root apices of 24. What is the most E. Papillomatosis likely diagnosis? 186. During preventive examination a A. Acute serous periodontitis patient was diagnosed with precancerous B. Acute suppurative periodontitis hyperkeratosis of the lower lip vermillion C. Acute suppurative pulpitis border. What treatment should be prescri- D. Acute diffuse pulpitis bed? E. Exacerbation of chronic periodontitis A. Surgical removal of the focus within 190. A 22-year-old man complains of tearing, healthy tissues throbbing, constant, intensifying pain in the B. Surgical removal of the focus within healthy tooth on the upper left jaw. The pain has been tissues + close-focus roentgenotherapy persisting for 4 days. Objectively tooth 26 has C. No treatment is necessary deep carious cavity non-communicating with D. Surgical removal of the focus within healthy the dental cavity.Probing is painless. Percussi- tissues + chemotherapy on is acutely painful. The tooth is mobile. E. Palliative treatment Mucogingival fold in the area of tooth 26 is painful on palpation. Make the diagnosis: 187. A 63-year-old man complains of fever and multiple painful rashes in his oral cavi- A. Acute suppurative periodontitis ty and on his face and torso. 3-4 days before B. Acute serous periodontitis the rashes appeared he had noticed a burni- C. Acute suppurative pulpitis ng sensation followed by sharp shooting D. Exacerbation of chronic periodontitis pain resembling that which occurs during E. Acute local pulpitis lumbago. The patient takes cytotoxic drugs for leukemia treatment. On examination 191. A 22-year-old patient has suffered uni- there are multiple aphthae located in a row lateral linear fracture in the area of the gonial on the vermillion border and labial, lingual, angle. Immobilization was provided with full and buccal mucosa on the right. The aphthae dental brace with loops and intermaxillary have hyperemic borders, they are not fused elastic expansion. Recovery was uncompli- together and are sharply painful on palpati- cated. The brace should be removed after: on. The right side of the face presents with erythematous spots, vesicles, and erosions. A. 3 weeks Make the provisional diagnosis: B. 2 weeks C. 1 week A. Herpes zoster D. 10 days B. Toxic allergic dermatostomatitis E. - C. Erythema multiforme exudativum D. Secondary syphilis 192. A 27-year-old patient was provisi- E. Chronic recurrent herpes onally diagnosed with acute suppurative odontogenic maxillary sinusitis. What radi- 188. A 34-year-old man presents with persisti- ology method would be the most informative ng dull pain in his tooth, which aggravates on in this case? biting. One week ago the tooth was treated for deep caries. Objectively on the masti- A. Computed tomography catory surface of tooth 36 there is a filling, B. X-ray percussion is painful, there is a supracontact C. Panoramic radiography observed in the area of 36. X-ray shows D. Spot-film radiography unchanged periodontium. What mistake was E. - made when tooth 36 was filled? 193. A 22-year-old man presents with swollen A. High filling and hyperemic mucosa of the retromolar B. Filling without insulation layer area; tooth 38 is covered with hood-shaped gingival flap that discharges pus on palpati- C. Gingival attachment is disturbed o D. Insulation layer exceeds borders of the on; body temperature is 37.5 C. What urgent dentinoenamel junction aid should be given to the patient in this case? E. Filling without medicinal substance sealed inside A. Gingival flap incision and antibacterial treatment 189. A 35-year-old man complains of persi- B. Gingival flap excision sting pain in tooth 24, which intensifies on C. Extraction of tooth 38 biting. Objectively on the distal masticatory D. treatment surface of tooth 24 there is a deep carious E. Gingival flap incision cavity filled with food debris. Percussion of cavity bottom is painless, there is no pain 194. A 43-year-old man came to the maxi- Krok 2 Stomatology (англомовний варiант, iноземнi студенти) 2018 рiк 24 llofacial surgeon with complaints of aesthetic neoplasm in the left parotid-masticatory regi- defect. Examination revealed excessive on. Skin over the tumor is without discolorati- accumulation of adipose tissue in the pati- on. The tumor is painless, but when the head ent’s neck and upper torso, which resembles bends down the tumor increases in size and collar with unclear magins; neck mobility assumes bluish coloring. What disease can be is reduced. Family history shows the same suspected in the child? symptoms to be present in the patient’s father. Make the provisional diagnosis: A. Hemangioma B. Fibroma A. Madelung’s deformity C. Atheroma B. Lipoma D. Lymphangioma C. Neurofibromatosis E. Cyst of the parotid gland D. Fibroma E. Lymphangioma 198. An 11-year-old child presents with protrusion of the upper front teeth, there 195. A 73-year-old man is registered for are tremata and diastema between the chi- regular check-ups in an oncological clinic ld’s teeth. What apparatus can be used for after completion of the combined treatment treatment of this pathology? for oral mucosa cancer stage II (radiation therapy and surgery). During one of the routi- A. Osadchy ne check-ups an area of exposed mandibular B. Brukle bone is detected. There are no inflammatory C. Frankel, type 3 changes of surrounding mucosa. A fistula D. Vasyl enko tract with soft granulation is detected. Mandi- E. Mershon bular X-ray shows a sequestrum without clear 199. A 40-year-old patient complains of pain margin between healthy and necrotic bone. in the tragus area, clicking during mouth What is the most likely provisional diagnosis? opening, stuffed . Objectively the face A. Mandibular is symmetrical, mouth opening path is strai- B. Acute purulent mandibular osteomyelitis ght. Dentition defect can be estimated as the C. Posttraumatic mandibular osteomyelitis I class by Kennedy; the 18, 17, 16, 26, 27, 28 D. Relapse of oral mucosa cancer teeth are missing. In this case the load would E. Chronic mandibular periostitis be the most traumatizing for the following anatomical structure: 196. A 19-year-old patient came to a dentofacial clinic with complaints of pain A. Interarticular disk in the gonial angle on the right, impaired B. Articular capsule mouth opening and painful chewing. The si- C. Articular head gns had been persisting for 5 days, emerged D. Distal slope of the articular tubercle spontaneously and had been aggravating E. Floor of the temporal bone socket gradually. Mandibular contracture is of the 200. A 38-year-old man complains of sensati- III degree. On examination of the oral cavity: on of a foreign body on his tongue and hyperemia, edema of the retromolar space on development of gag reflex during talking. the right, hood-shaped mucosa from under The signs appeared after the prolonged taki- which pus is being discharged and 2 tooth ng of . Objective examination tubercles can be detected. X-ray shows obli- detected thickened and pigmented filiform que medial tooth position. Make the di- papillae enlarged to 2-3 cm in size. Histologi- agnosis: cal analysis detected papillar hyperplasia and A. Acute suppurative of the 48 marked keratinization without alteration of tooth the surrounding tissues. What is the most li- B. Acute suppurative periostitis from the 48 kely diagnosis? tooth A. C. Chronic local mandibular osteomyelitis B. Median rhomboid D. Mandibular angle fracture C. Fissured tongue E. Acute submandibular sialadenitis D. Glossitis areata exfoliativa 197. Parents of a 6-year-old child complain E. of their child having a gradually enlarging

INSTRUCTIONAL BOOK

Ministry of public health of Ukraine (MPH of Ukraine) Department of human recources policy, education and science Testing Board

TEST ITEMS FOR LICENSING EXAMINATION: KROK 2. STOMATOLOGY.

Kyiv. Testing Board. (English language).

Approved to print 10.04./№32. Paper size 60х84 1/8 Offset paper. Typeface. Times New Roman Cyr. Offset print. Conditional print pages 24. Accounting publishing pages 28. Issue. 306 copies.

List of abbreviations

A/G Albumin/globulin ratio HR Heart rate A-ANON Alcoholics anonymous IDDM Insulin dependent diabetes mellitus ACT Abdominal computed tomography IFA Immunofluorescence assay ADP Adenosine diphosphate IHD Ischemic heart disease ALT Alanin aminotranspherase IU International unit AMP Adenosine monophosphate LDH Lactate dehydrogenase AP Action potential MSEC Medical and sanitary expert committee ARF Acute renal failure NAD Nicotine amide adenine dinucleotide AST Aspartat aminotranspherase NADPH Nicotine amide adenine dinucleotide phosphate restored ATP Adenosine triphosphate NIDDM Non-Insulin dependent diabetes mellitus BP Blood pressure PAC Polyunsaturated aromatic carbohydrates bpm Beats per minute PAS Periodic acid & shiff reaction

C.I. Color Index pCO2 CO2 partial pressure

CBC Complete blood count pO2 CO2 partial pressure CHF Chronic heart failure pm Per minute CT Computer tomography Ps Pulse rate DIC Disseminated intravascular coagualtion r Roentgen DCC Doctoral controlling committee RBC Red blood count DM-2 Non-Insulin dependent diabetes mellitus RDHA Reverse direct hemagglutination assay DTP Anti diphtheria-tetanus vaccine Rh Rhesus ECG Electrocardiogram (R)CFT Reiter's complement fixation test ESR Erythrocyte sedimentation rate RIHA Reverse indirect hemagglutination assay FC Function class RNA Ribonucleic acid FAD Flavin adenine dinucleotide RR Respiratory rate

FADH2 Flavin adenine dinucleotide restored S1 Heart sound 1 FEGDS Fibro-esphago-gastro-duodenoscopy S2 Heart sound 2

FMNH2 Flavin mononucleotide restored TU Tuberculin unit GIT U Unit Gy Gray USI Ultrasound investigation GMP Guanosine monophosphate V/f Vision field Hb Hemoglobin WBC White blood count HbA1c Glycosylated hemoglobin X-ray Roentgenogram Hct Hematocrit HIV Human immunodeficiency virus